Warum sollte das James-Webb-Weltraumteleskop im instabilen L2 bleiben?

Wir alle wissen, dass der Start des James-Webb-Weltraumteleskops für 2018 geplant ist. Es wurde entschieden, dass die Umlaufbahn von JWST elliptisch um den Lagrange-Punkt L2 sein wird , der als einer der instabilen Punkte (L1, L2, L3 ).

Warum also sollte es den Punkt umkreisen?

Ich habe ein paar Punkte. Die Instrumente des Teleskops sind ziemlich empfindlich und sollten immer von der Sonne (Wirklich? Ist das so?) und auch in einer kalten Umgebung ferngehalten werden - beides wird von L2 erfüllt. Wikipedia sagt dazu:

Die kombinierten Gravitationskräfte von Sonne und Erde können ein Raumschiff an diesem Punkt halten, so dass es theoretisch keinen Raketenschub braucht, um ein Raumschiff in der Umlaufbahn um L2 zu halten. In Wirklichkeit ist der stabile Punkt vergleichbar mit dem einer Kugel, die auf einer Sattelform balanciert. Entlang einer Richtung wird jede Störung den Ball zum stabilen Punkt treiben, während in der Querrichtung der Ball, wenn er gestört wird, vom stabilen Punkt wegfällt. Daher ist etwas Stationshaltung erforderlich, aber mit geringem Energieaufwand (nur 2–4 m/s pro Jahr, vom Gesamtbudget von 150 m/s)

Erstens, ist es überhaupt wirklich stabil? Wenn das in der Realität so instabil ist, warum sollte es dort positioniert werden? Ich meine, L4 & L5 Punkte sind in Ordnung. Warum kann das Teleskop nicht so positioniert werden, dass es immer von der Sonne nach außen zeigt? (Erde auch, wenn sie nicht wollen, dass die Reflexion von ihr selbst bricht)

@geoffc: "... so kommt die Sonne?" Deshalb habe ich "von der Sonne nach außen positioniert" erwähnt. IMHO denke ich, dass es machbar ist ;-)
@PearsonArtPhoto Ja, ich dachte eher an L3 als an L4/L5. Siehe en.wikipedia.org/wiki/Co-orbital_configuration Ignorieren Sie meinen Kommentar, können Sie ihn löschen.
@CrazyBuddy - in Bezug auf die Anforderung, Raumfahrzeuge von der Sonne fernzuhalten. Oh verdammt ja!! Das Teleskop wird laut Wikipedia bei „ungefähr 40 K (–233,2 °C; –387,7 °F)“ betrieben. Im Weltraum, in der Entfernung von der Sonne, wo JWST betrieben wird, würde direkte Sonneneinstrahlung es innerhalb weniger Minuten auf etwa 200 ° C aufheizen. Dies würde es unbrauchbar machen.
Völlig einverstanden, wir alle wissen wirklich, dass es 2018 auf den Markt kommt

Antworten (3)

Es gibt ein paar Gründe.

  1. Die Entfernung vom L2 zur Erde beträgt nur 1,5 Millionen km. Die L4/L5 sind 1 AE oder etwa 150 Millionen km entfernt. Das führt zu einer Verringerung der Verbindungsspanne von 40 db oder 1/10000. Das ist ziemlich bedeutsam. Um diesen Unterschied auszugleichen, benötigen Sie entweder eine größere Funkschüssel, mehr Leistung oder einen Datenverlust.
  2. Wie Sie bereits erwähnt haben, ist der Treibstoffverbrauch ziemlich gering, um diese Position zu halten, nur in der Größenordnung von 150 m/s delta v für die gesamte Mission. Das ist nicht viel und in der Tat weniger als nötig, um einen Satelliten im geostationären Orbit zu halten .
  3. Der Satellit ist viel näher, wodurch die Zeit zum Steuern eines Objekts verkürzt wird. Licht benötigt nur 5 Sekunden, um James Webb zu erreichen, während es 9 Minuten dauert, um L4/L5 zu erreichen. Dies schränkt die Fähigkeit ein, Echtzeitbefehle auszuführen, die gelegentlich nützlich sind (denken Sie an Gamma Ray Bursts, Super Novas usw.).

Unterm Strich wird das Kommunikationsproblem mit einem näheren Teleskop vereinfacht, und das macht mehr als wett, dass man etwas mehr Treibstoff mitnehmen muss.

... und wenn sie jemals etwas reparieren mussten, hypothetisch natürlich, roboterhaft oder bemannt, gelten auch hier die gleichen L2-Vorteile.

Ich glaube, es ist so, wie der Wikipedia-Unterartikel zu L2 sagt:

Die Sonne-Erde L2 ist ein guter Ort für weltraumgestützte Observatorien. Da ein Objekt um L2 die gleiche relative Position in Bezug auf Sonne und Erde beibehält, sind Abschirmung und Kalibrierung viel einfacher.

Dies gilt zwar, aber das Gleiche gilt für so ziemlich jeden der L-Punkte.
@PearsonArtPhoto- Abschirmung - L2 bleibt im Schatten der Erde, was bedeutet, dass die Sonne nicht blendet.
@SF., es gibt keinen Schatten im JW-Orbit - space.stackexchange.com/a/4111/2843 . Die Abschirmung ist einfacher als bei LEO, wenn sich ein sehr großes, warmes, IR-sendendes Objekt in zufälligen Winkeln um das Teleskop dreht - so viele Wärmeschocks pro Stunde, und es ist unmöglich, ein (aktiv und schnell rotierendes) EarthShield zu haben, um sich vor der Thermik zu verstecken Strahlung von der Erde für kryogene Spiegel. "Schatten" der Erde im Maßstab - en.wikipedia.org/wiki/Umbra,_penumbra_and_antumbra#Penumbra "Der volle Kegel erstreckt sich über 1,32 Millionen km.", und L2 beträgt 1,5 Millionen. Und die L2-Halo-Umlaufbahn ist 100 km vom L2-Punkt entfernt.
@osgx: das bedeutet nur keinen vollen Kernschatten. L2 bleibt in Antumbra , wodurch die Lichtmenge, die den Punkt erreicht, stark reduziert wird. Das ist definitiv anders als "kein Schatten" - anders als an jedem anderen Erde-Sonne-Lagrange-Punkt.
@SF. und immer noch befinden sich JWST (und andere Observatorien) in einer Halo-Umlaufbahn von 100 Kilometern und versuchen, ihren Weg so lange wie möglich von Antumbra und Penumbra fernzuhalten. Gaia - issfd.org/ISSFD_2014/ISSFD24_Paper_S2-5_Renk.pdf " Nachteil von Finsternissen ... Finsternisvermeidungsmanöver ... partielle Sonnenfinsternis ... ist unerwünscht in Bezug auf das thermische Gleichgewicht "; JWST - ntrs.nasa.gov/search.jsp?R=20160001318 " Erhebliche Anforderungen, die das JWST betreffen ... die Vermeidung von Erd-/Mondfinsternissen ... keine erlaubt, Einschränkungstreiber: Leistung und Thermik "

Zur Stabilität, L2 ist in radialer Richtung instabil : Wenn die Sonde etwas näher oder weiter in der Sonne-Erde-Achse ist, wird sie durch die Gravitation noch weiter geschoben.

L2 ist jedoch in der senkrechten Ebene stabil , weshalb Sie in einigen Animationen sehen, wie es L2 in der Ebene senkrecht zur Sonne-Erde-Achse umkreist: Die Gravitation wird es in Richtung L2 ziehen.

Siehe diesen Beitrag für eine detaillierte Erklärung.

Kennen Sie die dritte Richtung? Ich denke, ich würde es genau so formulieren. L2 verfolgt eine Umlaufbahn um die Sonne. Stellen Sie sich vor, der JWST befindet sich an einem Punkt auf dieser Umlaufbahn, der 1,5 Millionen km vom tatsächlichen L2-Punkt entfernt ist. Wird die Schwerkraft es in Richtung L2 ziehen? (Die Tatsache, dass es L2 umkreist, lässt mich denken, dass es sicherlich so sein muss, aber ich verstehe nicht wie.)
Tut mir leid, dass ich nicht genau bin :) Wenn ich "senkrechte Richtung" sage, meine ich "senkrechte Ebene". Also radiale Richtung = 1 instabile Richtung, senkrechte Ebene = 2 stabile Richtungen. Vergessen Sie auch nicht, wenn Sie über Stabilität sprechen, dass wir uns in einer rotierenden Referenz befinden. Die Schwerkraft wird das Raumschiff nicht alleine in Richtung L2 ziehen. Es ist eine Kombination aus Schwerkraft und Coriolis-Kräften, die Lagrange-Punkte bilden.